Difference between revisions of "1954 AHSME Problems/Problem 37"
Katzrockso (talk | contribs) (Created page with "== Problem 37== Given <math>\triangle PQR</math> with <math>\overline{RS}</math> bisecting <math>\angle R</math>, <math>PQ</math> extended to <math>D</math> and <math>\angle ...") |
|||
(8 intermediate revisions by 2 users not shown) | |||
Line 44: | Line 44: | ||
<math>\textbf{(A)}\ \angle m = \frac {1}{2}(\angle p - \angle q) \qquad | <math>\textbf{(A)}\ \angle m = \frac {1}{2}(\angle p - \angle q) \qquad | ||
− | \textbf{(B)}\ \angle m = \frac {1}{2}(\angle p + \angle q) | + | \textbf{(B)}\ \angle m = \frac {1}{2}(\angle p + \angle q) \qquad |
− | + | \textbf{(C)}\ \angle d =\frac{1}{2}(\angle q+\angle p)\qquad | |
\textbf{(D)}\ \angle d =\frac{1}{2}\angle m\qquad | \textbf{(D)}\ \angle d =\frac{1}{2}\angle m\qquad | ||
\textbf{(E)}\ \text{none of these is correct} </math> | \textbf{(E)}\ \text{none of these is correct} </math> | ||
+ | |||
+ | == Solution == | ||
+ | Let <math>\angle PRS</math> be <math>\theta</math>. | ||
+ | |||
+ | <math>p+ q + 2\theta = 180</math> | ||
+ | |||
+ | <math>m+\theta+90=180 \implies m+\theta=90 \implies 2m+2\theta=180</math> | ||
+ | |||
+ | <math>p+q+2\theta=2m+2\theta \implies \frac{p+q}{2}=m \implies \boxed{\textbf{(B) \ } \angle m = \frac{1}{2}(\angle p + \angle q)}</math> | ||
== Partial Solution == | == Partial Solution == | ||
<asy> | <asy> | ||
+ | import math; | ||
path anglemark2(pair A, pair B, pair C, real t=8, bool flip=false) | path anglemark2(pair A, pair B, pair C, real t=8, bool flip=false) | ||
{ | { | ||
Line 73: | Line 83: | ||
draw(R--Sp); | draw(R--Sp); | ||
draw(P--D--M); | draw(P--D--M); | ||
+ | pen f=fontsize(10pt); | ||
pair pI=extension(D,M,R,Q); | pair pI=extension(D,M,R,Q); | ||
− | + | label("$O$",pI+(-0.2,0.166),f); | |
draw(anglemark2(Sp,P,R,17)); | draw(anglemark2(Sp,P,R,17)); | ||
label("$p$",P+(0.35,0.1)); | label("$p$",P+(0.35,0.1)); | ||
Line 85: | Line 96: | ||
draw(anglemark2(M,D,P,29)); | draw(anglemark2(M,D,P,29)); | ||
label("$d$",D+(-0.75,0.095)); | label("$d$",D+(-0.75,0.095)); | ||
− | |||
label("$R$",R,N,f); | label("$R$",R,N,f); | ||
− | label("$M$",M+(-. | + | label("$M$",M+(-.07,.07),f); |
− | label("$N$",Np+(-. | + | label("$N$",Np+(-.08,.15),f); |
label("$P$",P,S,f); | label("$P$",P,S,f); | ||
label("$S$",Sp,S,f); | label("$S$",Sp,S,f); | ||
label("$Q$",Q,S,f); | label("$Q$",Q,S,f); | ||
label("$D$",D,S,f);</asy> | label("$D$",D,S,f);</asy> | ||
− | Looking at triangle PRQ, we have <math>\angle RPD+\angle RQS+\angle MRN=180</math> and from the given statement <math>\angle NMR=\frac{1}{2}\angle MRN</math>, so looking at triangle | + | Looking at triangle PRQ, we have <math>\angle RPD+\angle RQS+\angle MRN=180</math> and from the given statement <math>\angle NMR=\frac{1}{2}\angle MRN</math>, so looking at triangle MOR <math>\angle NMR=90-\frac{\angle RPD+\angle RQS}{2}</math>, which rules out |
+ | |||
+ | |||
+ | {{AHSME 50p box|year=1954|num-b=36|num-a=38}} | ||
+ | {{MAA Notice}} |
Latest revision as of 09:11, 3 May 2020
Problem 37
Given with bisecting , extended to and a right angle, then:
Solution
Let be .
Partial Solution
Looking at triangle PRQ, we have and from the given statement , so looking at triangle MOR , which rules out
1954 AHSC (Problems • Answer Key • Resources) | ||
Preceded by Problem 36 |
Followed by Problem 38 | |
1 • 2 • 3 • 4 • 5 • 6 • 7 • 8 • 9 • 10 • 11 • 12 • 13 • 14 • 15 • 16 • 17 • 18 • 19 • 20 • 21 • 22 • 23 • 24 • 25 • 26 • 27 • 28 • 29 • 30 • 31 • 32 • 33 • 34 • 35 • 36 • 37 • 38 • 39 • 40 • 41 • 42 • 43 • 44 • 45 • 46 • 47 • 48 • 49 • 50 | ||
All AHSME Problems and Solutions |
The problems on this page are copyrighted by the Mathematical Association of America's American Mathematics Competitions.